PT19.S4.Q09 - measurements of the motion of the planet uranus

Guadalajara180Guadalajara180 Alum Member
edited January 2016 in Logical Reasoning 14 karma
http://7sage.com/lsat_explanations/lsat-19-section-4-question-09/
Can someone explain why D is not the right answer? I was struggling between B and D but eventually chose D.

Comments

  • c.janson35c.janson35 Free Trial Inactive Sage Inactive ⭐
    2398 karma
    The argument concludes that there must exist some undiscovered planet because of the facts that (1) Uranus is being pulled away from the sun and (2) Neptune and Pluto do not have enough mass to pull Uranus away from the sun. But what if there was some other non-planetary body that was pulling Uranus away? Then the argument would be severely weakened. This is exactly what B says.

    D is incorrect because it does not account for the force pulling Uranus away, it only says the Sun's gravitational pull is weaker than its pull on the inner planets.

    Hope this helps!
  • Guadalajara180Guadalajara180 Alum Member
    14 karma
    @c.janson35 Thanks!! It makes a lot of sense now.
Sign In or Register to comment.